Find sum of series $ sum_{n=1}^{infty} (ncdot ln frac{2n+1}{2n-1} - 1) $












3












$begingroup$


how can I find sum of series $ sum_{n=1}^{infty} (ncdot ln frac{2n+1}{2n-1} - 1) $?
It is so weird for me because I put this to Mathematica and it tells me that sum does not converge...



Let consider sum no to infinity, but to n
$$ sum_{k=1}^{n} (kcdot ln frac{2k+1}{2k-1} - 1) =$$
$$ ln frac{3}{1}cdot left(frac{5}{3}right)^2 cdot...cdot left(frac{2n+1}{2n-1}right)^n - n = ln frac{1}{1}cdot frac{1}{3}cdot frac{1}{5}cdot ... frac{1}{2n-1} - n $$
but $$ n = ln e^n $$
so
it will be $$lnfrac{1}{e^n} cdot frac{1}{1}cdot frac{1}{3}cdot frac{1}{5}cdot ... frac{1}{2n-1}$$



So the limit of it is $-infty$
Have I done this well or I missed sth?










share|cite|improve this question











$endgroup$








  • 2




    $begingroup$
    it doesn't converge because the summand is around $nln(1+frac{2}{2n-1})-1 approx nfrac{2}{2n-1}-1 = frac{1}{2n-1}$.
    $endgroup$
    – mathworker21
    Nov 29 '18 at 19:08








  • 1




    $begingroup$
    It certainly converges, since the $n$'th term is asymptotic to $1/(12 n^2)$.
    $endgroup$
    – Robert Israel
    Nov 29 '18 at 19:09










  • $begingroup$
    @RobertIsrael where did I go wrong
    $endgroup$
    – mathworker21
    Nov 29 '18 at 19:14










  • $begingroup$
    @mathworker21 seems correct to me.
    $endgroup$
    – Connor Harris
    Nov 29 '18 at 19:16










  • $begingroup$
    where should it be?
    $endgroup$
    – mvxxx
    Nov 29 '18 at 19:28
















3












$begingroup$


how can I find sum of series $ sum_{n=1}^{infty} (ncdot ln frac{2n+1}{2n-1} - 1) $?
It is so weird for me because I put this to Mathematica and it tells me that sum does not converge...



Let consider sum no to infinity, but to n
$$ sum_{k=1}^{n} (kcdot ln frac{2k+1}{2k-1} - 1) =$$
$$ ln frac{3}{1}cdot left(frac{5}{3}right)^2 cdot...cdot left(frac{2n+1}{2n-1}right)^n - n = ln frac{1}{1}cdot frac{1}{3}cdot frac{1}{5}cdot ... frac{1}{2n-1} - n $$
but $$ n = ln e^n $$
so
it will be $$lnfrac{1}{e^n} cdot frac{1}{1}cdot frac{1}{3}cdot frac{1}{5}cdot ... frac{1}{2n-1}$$



So the limit of it is $-infty$
Have I done this well or I missed sth?










share|cite|improve this question











$endgroup$








  • 2




    $begingroup$
    it doesn't converge because the summand is around $nln(1+frac{2}{2n-1})-1 approx nfrac{2}{2n-1}-1 = frac{1}{2n-1}$.
    $endgroup$
    – mathworker21
    Nov 29 '18 at 19:08








  • 1




    $begingroup$
    It certainly converges, since the $n$'th term is asymptotic to $1/(12 n^2)$.
    $endgroup$
    – Robert Israel
    Nov 29 '18 at 19:09










  • $begingroup$
    @RobertIsrael where did I go wrong
    $endgroup$
    – mathworker21
    Nov 29 '18 at 19:14










  • $begingroup$
    @mathworker21 seems correct to me.
    $endgroup$
    – Connor Harris
    Nov 29 '18 at 19:16










  • $begingroup$
    where should it be?
    $endgroup$
    – mvxxx
    Nov 29 '18 at 19:28














3












3








3





$begingroup$


how can I find sum of series $ sum_{n=1}^{infty} (ncdot ln frac{2n+1}{2n-1} - 1) $?
It is so weird for me because I put this to Mathematica and it tells me that sum does not converge...



Let consider sum no to infinity, but to n
$$ sum_{k=1}^{n} (kcdot ln frac{2k+1}{2k-1} - 1) =$$
$$ ln frac{3}{1}cdot left(frac{5}{3}right)^2 cdot...cdot left(frac{2n+1}{2n-1}right)^n - n = ln frac{1}{1}cdot frac{1}{3}cdot frac{1}{5}cdot ... frac{1}{2n-1} - n $$
but $$ n = ln e^n $$
so
it will be $$lnfrac{1}{e^n} cdot frac{1}{1}cdot frac{1}{3}cdot frac{1}{5}cdot ... frac{1}{2n-1}$$



So the limit of it is $-infty$
Have I done this well or I missed sth?










share|cite|improve this question











$endgroup$




how can I find sum of series $ sum_{n=1}^{infty} (ncdot ln frac{2n+1}{2n-1} - 1) $?
It is so weird for me because I put this to Mathematica and it tells me that sum does not converge...



Let consider sum no to infinity, but to n
$$ sum_{k=1}^{n} (kcdot ln frac{2k+1}{2k-1} - 1) =$$
$$ ln frac{3}{1}cdot left(frac{5}{3}right)^2 cdot...cdot left(frac{2n+1}{2n-1}right)^n - n = ln frac{1}{1}cdot frac{1}{3}cdot frac{1}{5}cdot ... frac{1}{2n-1} - n $$
but $$ n = ln e^n $$
so
it will be $$lnfrac{1}{e^n} cdot frac{1}{1}cdot frac{1}{3}cdot frac{1}{5}cdot ... frac{1}{2n-1}$$



So the limit of it is $-infty$
Have I done this well or I missed sth?







real-analysis sequences-and-series






share|cite|improve this question















share|cite|improve this question













share|cite|improve this question




share|cite|improve this question








edited Nov 29 '18 at 19:25







mvxxx

















asked Nov 29 '18 at 19:06









mvxxxmvxxx

184




184








  • 2




    $begingroup$
    it doesn't converge because the summand is around $nln(1+frac{2}{2n-1})-1 approx nfrac{2}{2n-1}-1 = frac{1}{2n-1}$.
    $endgroup$
    – mathworker21
    Nov 29 '18 at 19:08








  • 1




    $begingroup$
    It certainly converges, since the $n$'th term is asymptotic to $1/(12 n^2)$.
    $endgroup$
    – Robert Israel
    Nov 29 '18 at 19:09










  • $begingroup$
    @RobertIsrael where did I go wrong
    $endgroup$
    – mathworker21
    Nov 29 '18 at 19:14










  • $begingroup$
    @mathworker21 seems correct to me.
    $endgroup$
    – Connor Harris
    Nov 29 '18 at 19:16










  • $begingroup$
    where should it be?
    $endgroup$
    – mvxxx
    Nov 29 '18 at 19:28














  • 2




    $begingroup$
    it doesn't converge because the summand is around $nln(1+frac{2}{2n-1})-1 approx nfrac{2}{2n-1}-1 = frac{1}{2n-1}$.
    $endgroup$
    – mathworker21
    Nov 29 '18 at 19:08








  • 1




    $begingroup$
    It certainly converges, since the $n$'th term is asymptotic to $1/(12 n^2)$.
    $endgroup$
    – Robert Israel
    Nov 29 '18 at 19:09










  • $begingroup$
    @RobertIsrael where did I go wrong
    $endgroup$
    – mathworker21
    Nov 29 '18 at 19:14










  • $begingroup$
    @mathworker21 seems correct to me.
    $endgroup$
    – Connor Harris
    Nov 29 '18 at 19:16










  • $begingroup$
    where should it be?
    $endgroup$
    – mvxxx
    Nov 29 '18 at 19:28








2




2




$begingroup$
it doesn't converge because the summand is around $nln(1+frac{2}{2n-1})-1 approx nfrac{2}{2n-1}-1 = frac{1}{2n-1}$.
$endgroup$
– mathworker21
Nov 29 '18 at 19:08






$begingroup$
it doesn't converge because the summand is around $nln(1+frac{2}{2n-1})-1 approx nfrac{2}{2n-1}-1 = frac{1}{2n-1}$.
$endgroup$
– mathworker21
Nov 29 '18 at 19:08






1




1




$begingroup$
It certainly converges, since the $n$'th term is asymptotic to $1/(12 n^2)$.
$endgroup$
– Robert Israel
Nov 29 '18 at 19:09




$begingroup$
It certainly converges, since the $n$'th term is asymptotic to $1/(12 n^2)$.
$endgroup$
– Robert Israel
Nov 29 '18 at 19:09












$begingroup$
@RobertIsrael where did I go wrong
$endgroup$
– mathworker21
Nov 29 '18 at 19:14




$begingroup$
@RobertIsrael where did I go wrong
$endgroup$
– mathworker21
Nov 29 '18 at 19:14












$begingroup$
@mathworker21 seems correct to me.
$endgroup$
– Connor Harris
Nov 29 '18 at 19:16




$begingroup$
@mathworker21 seems correct to me.
$endgroup$
– Connor Harris
Nov 29 '18 at 19:16












$begingroup$
where should it be?
$endgroup$
– mvxxx
Nov 29 '18 at 19:28




$begingroup$
where should it be?
$endgroup$
– mvxxx
Nov 29 '18 at 19:28










4 Answers
4






active

oldest

votes


















2












$begingroup$

We have that



$$sum_{n=1}^{N} left(ncdot ln frac{2n+1}{2n-1} - 1right)=sum_{n=1}^{N} left(ncdot ln (2n+1)-nln (2n-1) - 1right)=$$



$$=(1cdot ln 3-1cdot ln1-1)+(2cdot ln 5-2cdot ln3-1)+(3cdot ln 7-3cdot ln5-1)+ldots=$$



$$=-ln(3cdot 5cdot 7cdot ldotscdot (2N-1))+Ncdotln(2N+1)-N=$$$$=-lnleft(frac{(2N)!}{2^NN!}right)+Ncdotln(2N+1)-N=$$



$$=lnleft(frac{(2^N)^2N!N^N}{(2N)!e^N}right)+Ncdotlnleft(1+frac1{2N}right)$$



and by Stirling's approximation $N!sim sqrt{2pi N}left(frac{N}{e}right)^N$



$$frac{(2^N)^2N!N^N}{(2N!)e^N}simfrac{(2^N)^2N^N}{e^N}frac{sqrt{2pi N}}{sqrt{4pi N}}frac{N^Ne^{2N}}{e^N4^NN^{2N}}=frac{1}{sqrt 2}$$






share|cite|improve this answer











$endgroup$













  • $begingroup$
    I edited my post, can you look at this?
    $endgroup$
    – mvxxx
    Nov 29 '18 at 19:25










  • $begingroup$
    As noticed the limit exists since the term is asymthotic to $1/(12n^2)$. Starting by the first step indicated you see that the sum telescopes.
    $endgroup$
    – gimusi
    Nov 29 '18 at 19:28










  • $begingroup$
    @gimusi: Just was curious how did you precisely say the term is asymptotic to $1/(12n^2)$?
    $endgroup$
    – Yadati Kiran
    Nov 29 '18 at 19:36










  • $begingroup$
    @YadatiKiran It is not difficult to see by $ln frac{2n+1}{2n-1}=ln (1+1/2n)-ln (1-1/2n)$ and then using Taylor's series for log.
    $endgroup$
    – gimusi
    Nov 29 '18 at 19:46










  • $begingroup$
    @gimusi: Got it! Thanks.
    $endgroup$
    – Yadati Kiran
    Nov 29 '18 at 19:55



















2












$begingroup$

If I'm not mistaken, the actual sum is
$$ frac{1 - ln(2)}{2}$$






share|cite|improve this answer









$endgroup$













  • $begingroup$
    Yes that's agree with my result also.
    $endgroup$
    – gimusi
    Nov 29 '18 at 19:56



















2












$begingroup$

Let $f(x)=displaystylesum_{n=1}^{infty}left(nlnfrac{n+x}{n-x}-2xright)$ for $xin(-1,1)$. Then
$$f'(x)=displaystylesum_{n=1}^{infty}frac{2x^2}{n^2-x^2}=1-pi xcotpi x$$
(termwise differentiation is admissible because of uniform convergence of the latter series in $[-a,a]$ for any $0<a<1$; the second equality is known). Thus,
$$f(x)=x-frac{1}{pi}int_{0}^{pi x}tcot t,dt=x(1-lnsinpi x)+frac{1}{pi}int_{0}^{pi x}lnsin t,dt.$$
Your sum is $f(1/2)=(1-ln2)/2$.






share|cite|improve this answer









$endgroup$





















    1












    $begingroup$

    $newcommand{bbx}[1]{,bbox[15px,border:1px groove navy]{displaystyle{#1}},}
    newcommand{braces}[1]{leftlbrace,{#1},rightrbrace}
    newcommand{bracks}[1]{leftlbrack,{#1},rightrbrack}
    newcommand{dd}{mathrm{d}}
    newcommand{ds}[1]{displaystyle{#1}}
    newcommand{expo}[1]{,mathrm{e}^{#1},}
    newcommand{ic}{mathrm{i}}
    newcommand{mc}[1]{mathcal{#1}}
    newcommand{mrm}[1]{mathrm{#1}}
    newcommand{pars}[1]{left(,{#1},right)}
    newcommand{partiald}[3]{frac{partial^{#1} #2}{partial #3^{#1}}}
    newcommand{root}[2]{,sqrt[#1]{,{#2},},}
    newcommand{totald}[3]{frac{mathrm{d}^{#1} #2}{mathrm{d} #3^{#1}}}
    newcommand{verts}[1]{leftvert,{#1},rightvert}$




    With $ds{N in mathbb{N}_{geq 1}}$:




    begin{align}
    &bbox[#ffd,10px]{sum_{n = 1}^{N}bracks{nlnpars{2n + 1 over 2n - 1} - 1}}
    \[5mm] = &
    sum_{n = 1}^{N}nlnpars{2n + 1} - sum_{n = 1}^{N}nlnpars{2n - 1} - N
    \[5mm] = &
    -N + sum_{n = 0}^{N}nlnpars{2n + 1} -
    sum_{n = 0}^{N - 1}pars{n + 1}lnpars{2n + 1}
    \[5mm] = &
    -N + Nlnpars{2N + 1} -Nlnpars{2} -
    sum_{n = 0}^{N - 1}lnpars{n + {1 over 2}}
    \[5mm] = &
    -N + Nlnpars{N + {1 over 2}} -
    lnpars{prod_{n = 0}^{N - 1}bracks{n + {1 over 2}}}
    \[5mm] = &
    -N + Nlnpars{N + {1 over 2}} -
    lnpars{bracks{N - 1/2}! over Gammapars{1/2}}
    \[5mm] stackrel{mrm{as} N to infty}{sim}&
    -N + Nlnpars{N + {1 over 2}} -
    lnpars{root{2pi}bracks{N - 1/2}^{N}expo{-N + 1/2} over root{pi}}
    \[5mm] = &
    -N + Nlnpars{N + {1 over 2}} -
    lnpars{2^{1/2}N^{N}bracks{1 - {1/2 over N}}^{N}
    expo{-N + 1/2}}
    \[5mm] stackrel{mrm{as} N to infty}{sim} &
    -N + Nlnpars{N + {1 over 2}} -
    bracks{{1 over 2},lnpars{2} + Nlnpars{N} - N}
    \[5mm] = &
    underbrace{Nlnpars{1 + {1 over 2N}}}
    _{ds{stackrel{mrm{as} N to infty}{to} {1 over 2}}}
    - {1 over 2},lnpars{2}label{1}tag{1}
    \[5mm] stackrel{mrm{as} N to infty}{to} &
    bbx{1 - lnpars{2} over 2} approx 0.1534
    end{align}






    share|cite|improve this answer











    $endgroup$













    • $begingroup$
      How have you moved from one before last to last line?
      $endgroup$
      – mvxxx
      Nov 29 '18 at 21:16










    • $begingroup$
      @mvxxx I add one more line ( line (1) ) where you can see that the first $displaystyle -N$ cancels with the $displaystyle -left(-Nright)$ at the far right and $displaystyle Nlnleft(N + {1 over 2}right) - Nlnleft(Nright) = Nlnleft(1 + {1 over 2N}right) to {large{1 over 2}}$
      $endgroup$
      – Felix Marin
      Nov 29 '18 at 21:34













    Your Answer





    StackExchange.ifUsing("editor", function () {
    return StackExchange.using("mathjaxEditing", function () {
    StackExchange.MarkdownEditor.creationCallbacks.add(function (editor, postfix) {
    StackExchange.mathjaxEditing.prepareWmdForMathJax(editor, postfix, [["$", "$"], ["\\(","\\)"]]);
    });
    });
    }, "mathjax-editing");

    StackExchange.ready(function() {
    var channelOptions = {
    tags: "".split(" "),
    id: "69"
    };
    initTagRenderer("".split(" "), "".split(" "), channelOptions);

    StackExchange.using("externalEditor", function() {
    // Have to fire editor after snippets, if snippets enabled
    if (StackExchange.settings.snippets.snippetsEnabled) {
    StackExchange.using("snippets", function() {
    createEditor();
    });
    }
    else {
    createEditor();
    }
    });

    function createEditor() {
    StackExchange.prepareEditor({
    heartbeatType: 'answer',
    autoActivateHeartbeat: false,
    convertImagesToLinks: true,
    noModals: true,
    showLowRepImageUploadWarning: true,
    reputationToPostImages: 10,
    bindNavPrevention: true,
    postfix: "",
    imageUploader: {
    brandingHtml: "Powered by u003ca class="icon-imgur-white" href="https://imgur.com/"u003eu003c/au003e",
    contentPolicyHtml: "User contributions licensed under u003ca href="https://creativecommons.org/licenses/by-sa/3.0/"u003ecc by-sa 3.0 with attribution requiredu003c/au003e u003ca href="https://stackoverflow.com/legal/content-policy"u003e(content policy)u003c/au003e",
    allowUrls: true
    },
    noCode: true, onDemand: true,
    discardSelector: ".discard-answer"
    ,immediatelyShowMarkdownHelp:true
    });


    }
    });














    draft saved

    draft discarded


















    StackExchange.ready(
    function () {
    StackExchange.openid.initPostLogin('.new-post-login', 'https%3a%2f%2fmath.stackexchange.com%2fquestions%2f3019050%2ffind-sum-of-series-sum-n-1-infty-n-cdot-ln-frac2n12n-1-1%23new-answer', 'question_page');
    }
    );

    Post as a guest















    Required, but never shown

























    4 Answers
    4






    active

    oldest

    votes








    4 Answers
    4






    active

    oldest

    votes









    active

    oldest

    votes






    active

    oldest

    votes









    2












    $begingroup$

    We have that



    $$sum_{n=1}^{N} left(ncdot ln frac{2n+1}{2n-1} - 1right)=sum_{n=1}^{N} left(ncdot ln (2n+1)-nln (2n-1) - 1right)=$$



    $$=(1cdot ln 3-1cdot ln1-1)+(2cdot ln 5-2cdot ln3-1)+(3cdot ln 7-3cdot ln5-1)+ldots=$$



    $$=-ln(3cdot 5cdot 7cdot ldotscdot (2N-1))+Ncdotln(2N+1)-N=$$$$=-lnleft(frac{(2N)!}{2^NN!}right)+Ncdotln(2N+1)-N=$$



    $$=lnleft(frac{(2^N)^2N!N^N}{(2N)!e^N}right)+Ncdotlnleft(1+frac1{2N}right)$$



    and by Stirling's approximation $N!sim sqrt{2pi N}left(frac{N}{e}right)^N$



    $$frac{(2^N)^2N!N^N}{(2N!)e^N}simfrac{(2^N)^2N^N}{e^N}frac{sqrt{2pi N}}{sqrt{4pi N}}frac{N^Ne^{2N}}{e^N4^NN^{2N}}=frac{1}{sqrt 2}$$






    share|cite|improve this answer











    $endgroup$













    • $begingroup$
      I edited my post, can you look at this?
      $endgroup$
      – mvxxx
      Nov 29 '18 at 19:25










    • $begingroup$
      As noticed the limit exists since the term is asymthotic to $1/(12n^2)$. Starting by the first step indicated you see that the sum telescopes.
      $endgroup$
      – gimusi
      Nov 29 '18 at 19:28










    • $begingroup$
      @gimusi: Just was curious how did you precisely say the term is asymptotic to $1/(12n^2)$?
      $endgroup$
      – Yadati Kiran
      Nov 29 '18 at 19:36










    • $begingroup$
      @YadatiKiran It is not difficult to see by $ln frac{2n+1}{2n-1}=ln (1+1/2n)-ln (1-1/2n)$ and then using Taylor's series for log.
      $endgroup$
      – gimusi
      Nov 29 '18 at 19:46










    • $begingroup$
      @gimusi: Got it! Thanks.
      $endgroup$
      – Yadati Kiran
      Nov 29 '18 at 19:55
















    2












    $begingroup$

    We have that



    $$sum_{n=1}^{N} left(ncdot ln frac{2n+1}{2n-1} - 1right)=sum_{n=1}^{N} left(ncdot ln (2n+1)-nln (2n-1) - 1right)=$$



    $$=(1cdot ln 3-1cdot ln1-1)+(2cdot ln 5-2cdot ln3-1)+(3cdot ln 7-3cdot ln5-1)+ldots=$$



    $$=-ln(3cdot 5cdot 7cdot ldotscdot (2N-1))+Ncdotln(2N+1)-N=$$$$=-lnleft(frac{(2N)!}{2^NN!}right)+Ncdotln(2N+1)-N=$$



    $$=lnleft(frac{(2^N)^2N!N^N}{(2N)!e^N}right)+Ncdotlnleft(1+frac1{2N}right)$$



    and by Stirling's approximation $N!sim sqrt{2pi N}left(frac{N}{e}right)^N$



    $$frac{(2^N)^2N!N^N}{(2N!)e^N}simfrac{(2^N)^2N^N}{e^N}frac{sqrt{2pi N}}{sqrt{4pi N}}frac{N^Ne^{2N}}{e^N4^NN^{2N}}=frac{1}{sqrt 2}$$






    share|cite|improve this answer











    $endgroup$













    • $begingroup$
      I edited my post, can you look at this?
      $endgroup$
      – mvxxx
      Nov 29 '18 at 19:25










    • $begingroup$
      As noticed the limit exists since the term is asymthotic to $1/(12n^2)$. Starting by the first step indicated you see that the sum telescopes.
      $endgroup$
      – gimusi
      Nov 29 '18 at 19:28










    • $begingroup$
      @gimusi: Just was curious how did you precisely say the term is asymptotic to $1/(12n^2)$?
      $endgroup$
      – Yadati Kiran
      Nov 29 '18 at 19:36










    • $begingroup$
      @YadatiKiran It is not difficult to see by $ln frac{2n+1}{2n-1}=ln (1+1/2n)-ln (1-1/2n)$ and then using Taylor's series for log.
      $endgroup$
      – gimusi
      Nov 29 '18 at 19:46










    • $begingroup$
      @gimusi: Got it! Thanks.
      $endgroup$
      – Yadati Kiran
      Nov 29 '18 at 19:55














    2












    2








    2





    $begingroup$

    We have that



    $$sum_{n=1}^{N} left(ncdot ln frac{2n+1}{2n-1} - 1right)=sum_{n=1}^{N} left(ncdot ln (2n+1)-nln (2n-1) - 1right)=$$



    $$=(1cdot ln 3-1cdot ln1-1)+(2cdot ln 5-2cdot ln3-1)+(3cdot ln 7-3cdot ln5-1)+ldots=$$



    $$=-ln(3cdot 5cdot 7cdot ldotscdot (2N-1))+Ncdotln(2N+1)-N=$$$$=-lnleft(frac{(2N)!}{2^NN!}right)+Ncdotln(2N+1)-N=$$



    $$=lnleft(frac{(2^N)^2N!N^N}{(2N)!e^N}right)+Ncdotlnleft(1+frac1{2N}right)$$



    and by Stirling's approximation $N!sim sqrt{2pi N}left(frac{N}{e}right)^N$



    $$frac{(2^N)^2N!N^N}{(2N!)e^N}simfrac{(2^N)^2N^N}{e^N}frac{sqrt{2pi N}}{sqrt{4pi N}}frac{N^Ne^{2N}}{e^N4^NN^{2N}}=frac{1}{sqrt 2}$$






    share|cite|improve this answer











    $endgroup$



    We have that



    $$sum_{n=1}^{N} left(ncdot ln frac{2n+1}{2n-1} - 1right)=sum_{n=1}^{N} left(ncdot ln (2n+1)-nln (2n-1) - 1right)=$$



    $$=(1cdot ln 3-1cdot ln1-1)+(2cdot ln 5-2cdot ln3-1)+(3cdot ln 7-3cdot ln5-1)+ldots=$$



    $$=-ln(3cdot 5cdot 7cdot ldotscdot (2N-1))+Ncdotln(2N+1)-N=$$$$=-lnleft(frac{(2N)!}{2^NN!}right)+Ncdotln(2N+1)-N=$$



    $$=lnleft(frac{(2^N)^2N!N^N}{(2N)!e^N}right)+Ncdotlnleft(1+frac1{2N}right)$$



    and by Stirling's approximation $N!sim sqrt{2pi N}left(frac{N}{e}right)^N$



    $$frac{(2^N)^2N!N^N}{(2N!)e^N}simfrac{(2^N)^2N^N}{e^N}frac{sqrt{2pi N}}{sqrt{4pi N}}frac{N^Ne^{2N}}{e^N4^NN^{2N}}=frac{1}{sqrt 2}$$







    share|cite|improve this answer














    share|cite|improve this answer



    share|cite|improve this answer








    edited Nov 29 '18 at 21:30

























    answered Nov 29 '18 at 19:21









    gimusigimusi

    92.8k84494




    92.8k84494












    • $begingroup$
      I edited my post, can you look at this?
      $endgroup$
      – mvxxx
      Nov 29 '18 at 19:25










    • $begingroup$
      As noticed the limit exists since the term is asymthotic to $1/(12n^2)$. Starting by the first step indicated you see that the sum telescopes.
      $endgroup$
      – gimusi
      Nov 29 '18 at 19:28










    • $begingroup$
      @gimusi: Just was curious how did you precisely say the term is asymptotic to $1/(12n^2)$?
      $endgroup$
      – Yadati Kiran
      Nov 29 '18 at 19:36










    • $begingroup$
      @YadatiKiran It is not difficult to see by $ln frac{2n+1}{2n-1}=ln (1+1/2n)-ln (1-1/2n)$ and then using Taylor's series for log.
      $endgroup$
      – gimusi
      Nov 29 '18 at 19:46










    • $begingroup$
      @gimusi: Got it! Thanks.
      $endgroup$
      – Yadati Kiran
      Nov 29 '18 at 19:55


















    • $begingroup$
      I edited my post, can you look at this?
      $endgroup$
      – mvxxx
      Nov 29 '18 at 19:25










    • $begingroup$
      As noticed the limit exists since the term is asymthotic to $1/(12n^2)$. Starting by the first step indicated you see that the sum telescopes.
      $endgroup$
      – gimusi
      Nov 29 '18 at 19:28










    • $begingroup$
      @gimusi: Just was curious how did you precisely say the term is asymptotic to $1/(12n^2)$?
      $endgroup$
      – Yadati Kiran
      Nov 29 '18 at 19:36










    • $begingroup$
      @YadatiKiran It is not difficult to see by $ln frac{2n+1}{2n-1}=ln (1+1/2n)-ln (1-1/2n)$ and then using Taylor's series for log.
      $endgroup$
      – gimusi
      Nov 29 '18 at 19:46










    • $begingroup$
      @gimusi: Got it! Thanks.
      $endgroup$
      – Yadati Kiran
      Nov 29 '18 at 19:55
















    $begingroup$
    I edited my post, can you look at this?
    $endgroup$
    – mvxxx
    Nov 29 '18 at 19:25




    $begingroup$
    I edited my post, can you look at this?
    $endgroup$
    – mvxxx
    Nov 29 '18 at 19:25












    $begingroup$
    As noticed the limit exists since the term is asymthotic to $1/(12n^2)$. Starting by the first step indicated you see that the sum telescopes.
    $endgroup$
    – gimusi
    Nov 29 '18 at 19:28




    $begingroup$
    As noticed the limit exists since the term is asymthotic to $1/(12n^2)$. Starting by the first step indicated you see that the sum telescopes.
    $endgroup$
    – gimusi
    Nov 29 '18 at 19:28












    $begingroup$
    @gimusi: Just was curious how did you precisely say the term is asymptotic to $1/(12n^2)$?
    $endgroup$
    – Yadati Kiran
    Nov 29 '18 at 19:36




    $begingroup$
    @gimusi: Just was curious how did you precisely say the term is asymptotic to $1/(12n^2)$?
    $endgroup$
    – Yadati Kiran
    Nov 29 '18 at 19:36












    $begingroup$
    @YadatiKiran It is not difficult to see by $ln frac{2n+1}{2n-1}=ln (1+1/2n)-ln (1-1/2n)$ and then using Taylor's series for log.
    $endgroup$
    – gimusi
    Nov 29 '18 at 19:46




    $begingroup$
    @YadatiKiran It is not difficult to see by $ln frac{2n+1}{2n-1}=ln (1+1/2n)-ln (1-1/2n)$ and then using Taylor's series for log.
    $endgroup$
    – gimusi
    Nov 29 '18 at 19:46












    $begingroup$
    @gimusi: Got it! Thanks.
    $endgroup$
    – Yadati Kiran
    Nov 29 '18 at 19:55




    $begingroup$
    @gimusi: Got it! Thanks.
    $endgroup$
    – Yadati Kiran
    Nov 29 '18 at 19:55











    2












    $begingroup$

    If I'm not mistaken, the actual sum is
    $$ frac{1 - ln(2)}{2}$$






    share|cite|improve this answer









    $endgroup$













    • $begingroup$
      Yes that's agree with my result also.
      $endgroup$
      – gimusi
      Nov 29 '18 at 19:56
















    2












    $begingroup$

    If I'm not mistaken, the actual sum is
    $$ frac{1 - ln(2)}{2}$$






    share|cite|improve this answer









    $endgroup$













    • $begingroup$
      Yes that's agree with my result also.
      $endgroup$
      – gimusi
      Nov 29 '18 at 19:56














    2












    2








    2





    $begingroup$

    If I'm not mistaken, the actual sum is
    $$ frac{1 - ln(2)}{2}$$






    share|cite|improve this answer









    $endgroup$



    If I'm not mistaken, the actual sum is
    $$ frac{1 - ln(2)}{2}$$







    share|cite|improve this answer












    share|cite|improve this answer



    share|cite|improve this answer










    answered Nov 29 '18 at 19:42









    Robert IsraelRobert Israel

    322k23212465




    322k23212465












    • $begingroup$
      Yes that's agree with my result also.
      $endgroup$
      – gimusi
      Nov 29 '18 at 19:56


















    • $begingroup$
      Yes that's agree with my result also.
      $endgroup$
      – gimusi
      Nov 29 '18 at 19:56
















    $begingroup$
    Yes that's agree with my result also.
    $endgroup$
    – gimusi
    Nov 29 '18 at 19:56




    $begingroup$
    Yes that's agree with my result also.
    $endgroup$
    – gimusi
    Nov 29 '18 at 19:56











    2












    $begingroup$

    Let $f(x)=displaystylesum_{n=1}^{infty}left(nlnfrac{n+x}{n-x}-2xright)$ for $xin(-1,1)$. Then
    $$f'(x)=displaystylesum_{n=1}^{infty}frac{2x^2}{n^2-x^2}=1-pi xcotpi x$$
    (termwise differentiation is admissible because of uniform convergence of the latter series in $[-a,a]$ for any $0<a<1$; the second equality is known). Thus,
    $$f(x)=x-frac{1}{pi}int_{0}^{pi x}tcot t,dt=x(1-lnsinpi x)+frac{1}{pi}int_{0}^{pi x}lnsin t,dt.$$
    Your sum is $f(1/2)=(1-ln2)/2$.






    share|cite|improve this answer









    $endgroup$


















      2












      $begingroup$

      Let $f(x)=displaystylesum_{n=1}^{infty}left(nlnfrac{n+x}{n-x}-2xright)$ for $xin(-1,1)$. Then
      $$f'(x)=displaystylesum_{n=1}^{infty}frac{2x^2}{n^2-x^2}=1-pi xcotpi x$$
      (termwise differentiation is admissible because of uniform convergence of the latter series in $[-a,a]$ for any $0<a<1$; the second equality is known). Thus,
      $$f(x)=x-frac{1}{pi}int_{0}^{pi x}tcot t,dt=x(1-lnsinpi x)+frac{1}{pi}int_{0}^{pi x}lnsin t,dt.$$
      Your sum is $f(1/2)=(1-ln2)/2$.






      share|cite|improve this answer









      $endgroup$
















        2












        2








        2





        $begingroup$

        Let $f(x)=displaystylesum_{n=1}^{infty}left(nlnfrac{n+x}{n-x}-2xright)$ for $xin(-1,1)$. Then
        $$f'(x)=displaystylesum_{n=1}^{infty}frac{2x^2}{n^2-x^2}=1-pi xcotpi x$$
        (termwise differentiation is admissible because of uniform convergence of the latter series in $[-a,a]$ for any $0<a<1$; the second equality is known). Thus,
        $$f(x)=x-frac{1}{pi}int_{0}^{pi x}tcot t,dt=x(1-lnsinpi x)+frac{1}{pi}int_{0}^{pi x}lnsin t,dt.$$
        Your sum is $f(1/2)=(1-ln2)/2$.






        share|cite|improve this answer









        $endgroup$



        Let $f(x)=displaystylesum_{n=1}^{infty}left(nlnfrac{n+x}{n-x}-2xright)$ for $xin(-1,1)$. Then
        $$f'(x)=displaystylesum_{n=1}^{infty}frac{2x^2}{n^2-x^2}=1-pi xcotpi x$$
        (termwise differentiation is admissible because of uniform convergence of the latter series in $[-a,a]$ for any $0<a<1$; the second equality is known). Thus,
        $$f(x)=x-frac{1}{pi}int_{0}^{pi x}tcot t,dt=x(1-lnsinpi x)+frac{1}{pi}int_{0}^{pi x}lnsin t,dt.$$
        Your sum is $f(1/2)=(1-ln2)/2$.







        share|cite|improve this answer












        share|cite|improve this answer



        share|cite|improve this answer










        answered Nov 29 '18 at 19:53









        metamorphymetamorphy

        3,6821621




        3,6821621























            1












            $begingroup$

            $newcommand{bbx}[1]{,bbox[15px,border:1px groove navy]{displaystyle{#1}},}
            newcommand{braces}[1]{leftlbrace,{#1},rightrbrace}
            newcommand{bracks}[1]{leftlbrack,{#1},rightrbrack}
            newcommand{dd}{mathrm{d}}
            newcommand{ds}[1]{displaystyle{#1}}
            newcommand{expo}[1]{,mathrm{e}^{#1},}
            newcommand{ic}{mathrm{i}}
            newcommand{mc}[1]{mathcal{#1}}
            newcommand{mrm}[1]{mathrm{#1}}
            newcommand{pars}[1]{left(,{#1},right)}
            newcommand{partiald}[3]{frac{partial^{#1} #2}{partial #3^{#1}}}
            newcommand{root}[2]{,sqrt[#1]{,{#2},},}
            newcommand{totald}[3]{frac{mathrm{d}^{#1} #2}{mathrm{d} #3^{#1}}}
            newcommand{verts}[1]{leftvert,{#1},rightvert}$




            With $ds{N in mathbb{N}_{geq 1}}$:




            begin{align}
            &bbox[#ffd,10px]{sum_{n = 1}^{N}bracks{nlnpars{2n + 1 over 2n - 1} - 1}}
            \[5mm] = &
            sum_{n = 1}^{N}nlnpars{2n + 1} - sum_{n = 1}^{N}nlnpars{2n - 1} - N
            \[5mm] = &
            -N + sum_{n = 0}^{N}nlnpars{2n + 1} -
            sum_{n = 0}^{N - 1}pars{n + 1}lnpars{2n + 1}
            \[5mm] = &
            -N + Nlnpars{2N + 1} -Nlnpars{2} -
            sum_{n = 0}^{N - 1}lnpars{n + {1 over 2}}
            \[5mm] = &
            -N + Nlnpars{N + {1 over 2}} -
            lnpars{prod_{n = 0}^{N - 1}bracks{n + {1 over 2}}}
            \[5mm] = &
            -N + Nlnpars{N + {1 over 2}} -
            lnpars{bracks{N - 1/2}! over Gammapars{1/2}}
            \[5mm] stackrel{mrm{as} N to infty}{sim}&
            -N + Nlnpars{N + {1 over 2}} -
            lnpars{root{2pi}bracks{N - 1/2}^{N}expo{-N + 1/2} over root{pi}}
            \[5mm] = &
            -N + Nlnpars{N + {1 over 2}} -
            lnpars{2^{1/2}N^{N}bracks{1 - {1/2 over N}}^{N}
            expo{-N + 1/2}}
            \[5mm] stackrel{mrm{as} N to infty}{sim} &
            -N + Nlnpars{N + {1 over 2}} -
            bracks{{1 over 2},lnpars{2} + Nlnpars{N} - N}
            \[5mm] = &
            underbrace{Nlnpars{1 + {1 over 2N}}}
            _{ds{stackrel{mrm{as} N to infty}{to} {1 over 2}}}
            - {1 over 2},lnpars{2}label{1}tag{1}
            \[5mm] stackrel{mrm{as} N to infty}{to} &
            bbx{1 - lnpars{2} over 2} approx 0.1534
            end{align}






            share|cite|improve this answer











            $endgroup$













            • $begingroup$
              How have you moved from one before last to last line?
              $endgroup$
              – mvxxx
              Nov 29 '18 at 21:16










            • $begingroup$
              @mvxxx I add one more line ( line (1) ) where you can see that the first $displaystyle -N$ cancels with the $displaystyle -left(-Nright)$ at the far right and $displaystyle Nlnleft(N + {1 over 2}right) - Nlnleft(Nright) = Nlnleft(1 + {1 over 2N}right) to {large{1 over 2}}$
              $endgroup$
              – Felix Marin
              Nov 29 '18 at 21:34


















            1












            $begingroup$

            $newcommand{bbx}[1]{,bbox[15px,border:1px groove navy]{displaystyle{#1}},}
            newcommand{braces}[1]{leftlbrace,{#1},rightrbrace}
            newcommand{bracks}[1]{leftlbrack,{#1},rightrbrack}
            newcommand{dd}{mathrm{d}}
            newcommand{ds}[1]{displaystyle{#1}}
            newcommand{expo}[1]{,mathrm{e}^{#1},}
            newcommand{ic}{mathrm{i}}
            newcommand{mc}[1]{mathcal{#1}}
            newcommand{mrm}[1]{mathrm{#1}}
            newcommand{pars}[1]{left(,{#1},right)}
            newcommand{partiald}[3]{frac{partial^{#1} #2}{partial #3^{#1}}}
            newcommand{root}[2]{,sqrt[#1]{,{#2},},}
            newcommand{totald}[3]{frac{mathrm{d}^{#1} #2}{mathrm{d} #3^{#1}}}
            newcommand{verts}[1]{leftvert,{#1},rightvert}$




            With $ds{N in mathbb{N}_{geq 1}}$:




            begin{align}
            &bbox[#ffd,10px]{sum_{n = 1}^{N}bracks{nlnpars{2n + 1 over 2n - 1} - 1}}
            \[5mm] = &
            sum_{n = 1}^{N}nlnpars{2n + 1} - sum_{n = 1}^{N}nlnpars{2n - 1} - N
            \[5mm] = &
            -N + sum_{n = 0}^{N}nlnpars{2n + 1} -
            sum_{n = 0}^{N - 1}pars{n + 1}lnpars{2n + 1}
            \[5mm] = &
            -N + Nlnpars{2N + 1} -Nlnpars{2} -
            sum_{n = 0}^{N - 1}lnpars{n + {1 over 2}}
            \[5mm] = &
            -N + Nlnpars{N + {1 over 2}} -
            lnpars{prod_{n = 0}^{N - 1}bracks{n + {1 over 2}}}
            \[5mm] = &
            -N + Nlnpars{N + {1 over 2}} -
            lnpars{bracks{N - 1/2}! over Gammapars{1/2}}
            \[5mm] stackrel{mrm{as} N to infty}{sim}&
            -N + Nlnpars{N + {1 over 2}} -
            lnpars{root{2pi}bracks{N - 1/2}^{N}expo{-N + 1/2} over root{pi}}
            \[5mm] = &
            -N + Nlnpars{N + {1 over 2}} -
            lnpars{2^{1/2}N^{N}bracks{1 - {1/2 over N}}^{N}
            expo{-N + 1/2}}
            \[5mm] stackrel{mrm{as} N to infty}{sim} &
            -N + Nlnpars{N + {1 over 2}} -
            bracks{{1 over 2},lnpars{2} + Nlnpars{N} - N}
            \[5mm] = &
            underbrace{Nlnpars{1 + {1 over 2N}}}
            _{ds{stackrel{mrm{as} N to infty}{to} {1 over 2}}}
            - {1 over 2},lnpars{2}label{1}tag{1}
            \[5mm] stackrel{mrm{as} N to infty}{to} &
            bbx{1 - lnpars{2} over 2} approx 0.1534
            end{align}






            share|cite|improve this answer











            $endgroup$













            • $begingroup$
              How have you moved from one before last to last line?
              $endgroup$
              – mvxxx
              Nov 29 '18 at 21:16










            • $begingroup$
              @mvxxx I add one more line ( line (1) ) where you can see that the first $displaystyle -N$ cancels with the $displaystyle -left(-Nright)$ at the far right and $displaystyle Nlnleft(N + {1 over 2}right) - Nlnleft(Nright) = Nlnleft(1 + {1 over 2N}right) to {large{1 over 2}}$
              $endgroup$
              – Felix Marin
              Nov 29 '18 at 21:34
















            1












            1








            1





            $begingroup$

            $newcommand{bbx}[1]{,bbox[15px,border:1px groove navy]{displaystyle{#1}},}
            newcommand{braces}[1]{leftlbrace,{#1},rightrbrace}
            newcommand{bracks}[1]{leftlbrack,{#1},rightrbrack}
            newcommand{dd}{mathrm{d}}
            newcommand{ds}[1]{displaystyle{#1}}
            newcommand{expo}[1]{,mathrm{e}^{#1},}
            newcommand{ic}{mathrm{i}}
            newcommand{mc}[1]{mathcal{#1}}
            newcommand{mrm}[1]{mathrm{#1}}
            newcommand{pars}[1]{left(,{#1},right)}
            newcommand{partiald}[3]{frac{partial^{#1} #2}{partial #3^{#1}}}
            newcommand{root}[2]{,sqrt[#1]{,{#2},},}
            newcommand{totald}[3]{frac{mathrm{d}^{#1} #2}{mathrm{d} #3^{#1}}}
            newcommand{verts}[1]{leftvert,{#1},rightvert}$




            With $ds{N in mathbb{N}_{geq 1}}$:




            begin{align}
            &bbox[#ffd,10px]{sum_{n = 1}^{N}bracks{nlnpars{2n + 1 over 2n - 1} - 1}}
            \[5mm] = &
            sum_{n = 1}^{N}nlnpars{2n + 1} - sum_{n = 1}^{N}nlnpars{2n - 1} - N
            \[5mm] = &
            -N + sum_{n = 0}^{N}nlnpars{2n + 1} -
            sum_{n = 0}^{N - 1}pars{n + 1}lnpars{2n + 1}
            \[5mm] = &
            -N + Nlnpars{2N + 1} -Nlnpars{2} -
            sum_{n = 0}^{N - 1}lnpars{n + {1 over 2}}
            \[5mm] = &
            -N + Nlnpars{N + {1 over 2}} -
            lnpars{prod_{n = 0}^{N - 1}bracks{n + {1 over 2}}}
            \[5mm] = &
            -N + Nlnpars{N + {1 over 2}} -
            lnpars{bracks{N - 1/2}! over Gammapars{1/2}}
            \[5mm] stackrel{mrm{as} N to infty}{sim}&
            -N + Nlnpars{N + {1 over 2}} -
            lnpars{root{2pi}bracks{N - 1/2}^{N}expo{-N + 1/2} over root{pi}}
            \[5mm] = &
            -N + Nlnpars{N + {1 over 2}} -
            lnpars{2^{1/2}N^{N}bracks{1 - {1/2 over N}}^{N}
            expo{-N + 1/2}}
            \[5mm] stackrel{mrm{as} N to infty}{sim} &
            -N + Nlnpars{N + {1 over 2}} -
            bracks{{1 over 2},lnpars{2} + Nlnpars{N} - N}
            \[5mm] = &
            underbrace{Nlnpars{1 + {1 over 2N}}}
            _{ds{stackrel{mrm{as} N to infty}{to} {1 over 2}}}
            - {1 over 2},lnpars{2}label{1}tag{1}
            \[5mm] stackrel{mrm{as} N to infty}{to} &
            bbx{1 - lnpars{2} over 2} approx 0.1534
            end{align}






            share|cite|improve this answer











            $endgroup$



            $newcommand{bbx}[1]{,bbox[15px,border:1px groove navy]{displaystyle{#1}},}
            newcommand{braces}[1]{leftlbrace,{#1},rightrbrace}
            newcommand{bracks}[1]{leftlbrack,{#1},rightrbrack}
            newcommand{dd}{mathrm{d}}
            newcommand{ds}[1]{displaystyle{#1}}
            newcommand{expo}[1]{,mathrm{e}^{#1},}
            newcommand{ic}{mathrm{i}}
            newcommand{mc}[1]{mathcal{#1}}
            newcommand{mrm}[1]{mathrm{#1}}
            newcommand{pars}[1]{left(,{#1},right)}
            newcommand{partiald}[3]{frac{partial^{#1} #2}{partial #3^{#1}}}
            newcommand{root}[2]{,sqrt[#1]{,{#2},},}
            newcommand{totald}[3]{frac{mathrm{d}^{#1} #2}{mathrm{d} #3^{#1}}}
            newcommand{verts}[1]{leftvert,{#1},rightvert}$




            With $ds{N in mathbb{N}_{geq 1}}$:




            begin{align}
            &bbox[#ffd,10px]{sum_{n = 1}^{N}bracks{nlnpars{2n + 1 over 2n - 1} - 1}}
            \[5mm] = &
            sum_{n = 1}^{N}nlnpars{2n + 1} - sum_{n = 1}^{N}nlnpars{2n - 1} - N
            \[5mm] = &
            -N + sum_{n = 0}^{N}nlnpars{2n + 1} -
            sum_{n = 0}^{N - 1}pars{n + 1}lnpars{2n + 1}
            \[5mm] = &
            -N + Nlnpars{2N + 1} -Nlnpars{2} -
            sum_{n = 0}^{N - 1}lnpars{n + {1 over 2}}
            \[5mm] = &
            -N + Nlnpars{N + {1 over 2}} -
            lnpars{prod_{n = 0}^{N - 1}bracks{n + {1 over 2}}}
            \[5mm] = &
            -N + Nlnpars{N + {1 over 2}} -
            lnpars{bracks{N - 1/2}! over Gammapars{1/2}}
            \[5mm] stackrel{mrm{as} N to infty}{sim}&
            -N + Nlnpars{N + {1 over 2}} -
            lnpars{root{2pi}bracks{N - 1/2}^{N}expo{-N + 1/2} over root{pi}}
            \[5mm] = &
            -N + Nlnpars{N + {1 over 2}} -
            lnpars{2^{1/2}N^{N}bracks{1 - {1/2 over N}}^{N}
            expo{-N + 1/2}}
            \[5mm] stackrel{mrm{as} N to infty}{sim} &
            -N + Nlnpars{N + {1 over 2}} -
            bracks{{1 over 2},lnpars{2} + Nlnpars{N} - N}
            \[5mm] = &
            underbrace{Nlnpars{1 + {1 over 2N}}}
            _{ds{stackrel{mrm{as} N to infty}{to} {1 over 2}}}
            - {1 over 2},lnpars{2}label{1}tag{1}
            \[5mm] stackrel{mrm{as} N to infty}{to} &
            bbx{1 - lnpars{2} over 2} approx 0.1534
            end{align}







            share|cite|improve this answer














            share|cite|improve this answer



            share|cite|improve this answer








            edited Nov 29 '18 at 21:30

























            answered Nov 29 '18 at 21:00









            Felix MarinFelix Marin

            67.8k7107142




            67.8k7107142












            • $begingroup$
              How have you moved from one before last to last line?
              $endgroup$
              – mvxxx
              Nov 29 '18 at 21:16










            • $begingroup$
              @mvxxx I add one more line ( line (1) ) where you can see that the first $displaystyle -N$ cancels with the $displaystyle -left(-Nright)$ at the far right and $displaystyle Nlnleft(N + {1 over 2}right) - Nlnleft(Nright) = Nlnleft(1 + {1 over 2N}right) to {large{1 over 2}}$
              $endgroup$
              – Felix Marin
              Nov 29 '18 at 21:34




















            • $begingroup$
              How have you moved from one before last to last line?
              $endgroup$
              – mvxxx
              Nov 29 '18 at 21:16










            • $begingroup$
              @mvxxx I add one more line ( line (1) ) where you can see that the first $displaystyle -N$ cancels with the $displaystyle -left(-Nright)$ at the far right and $displaystyle Nlnleft(N + {1 over 2}right) - Nlnleft(Nright) = Nlnleft(1 + {1 over 2N}right) to {large{1 over 2}}$
              $endgroup$
              – Felix Marin
              Nov 29 '18 at 21:34


















            $begingroup$
            How have you moved from one before last to last line?
            $endgroup$
            – mvxxx
            Nov 29 '18 at 21:16




            $begingroup$
            How have you moved from one before last to last line?
            $endgroup$
            – mvxxx
            Nov 29 '18 at 21:16












            $begingroup$
            @mvxxx I add one more line ( line (1) ) where you can see that the first $displaystyle -N$ cancels with the $displaystyle -left(-Nright)$ at the far right and $displaystyle Nlnleft(N + {1 over 2}right) - Nlnleft(Nright) = Nlnleft(1 + {1 over 2N}right) to {large{1 over 2}}$
            $endgroup$
            – Felix Marin
            Nov 29 '18 at 21:34






            $begingroup$
            @mvxxx I add one more line ( line (1) ) where you can see that the first $displaystyle -N$ cancels with the $displaystyle -left(-Nright)$ at the far right and $displaystyle Nlnleft(N + {1 over 2}right) - Nlnleft(Nright) = Nlnleft(1 + {1 over 2N}right) to {large{1 over 2}}$
            $endgroup$
            – Felix Marin
            Nov 29 '18 at 21:34




















            draft saved

            draft discarded




















































            Thanks for contributing an answer to Mathematics Stack Exchange!


            • Please be sure to answer the question. Provide details and share your research!

            But avoid



            • Asking for help, clarification, or responding to other answers.

            • Making statements based on opinion; back them up with references or personal experience.


            Use MathJax to format equations. MathJax reference.


            To learn more, see our tips on writing great answers.




            draft saved


            draft discarded














            StackExchange.ready(
            function () {
            StackExchange.openid.initPostLogin('.new-post-login', 'https%3a%2f%2fmath.stackexchange.com%2fquestions%2f3019050%2ffind-sum-of-series-sum-n-1-infty-n-cdot-ln-frac2n12n-1-1%23new-answer', 'question_page');
            }
            );

            Post as a guest















            Required, but never shown





















































            Required, but never shown














            Required, but never shown












            Required, but never shown







            Required, but never shown

































            Required, but never shown














            Required, but never shown












            Required, but never shown







            Required, but never shown







            Popular posts from this blog

            Biblatex bibliography style without URLs when DOI exists (in Overleaf with Zotero bibliography)

            ComboBox Display Member on multiple fields

            Is it possible to collect Nectar points via Trainline?